Keep this for future reference.
Answer:
1a. corresponding angles
1b. alternate exterior angles
1c. same side interior angles
2. it is simple, solve for x just like in Algebra, x=9
4x+2=7x-25
4x+27=7x
27=3x
9=x
Step-by-step explanation:
What do I put in the boxes if the answer is 1.74%?
If a credit card advertises an annual interest rate of 23%, then the equivalent monthly interest rate is 1.74%
The annual interest rate = 23%
The credit card interest rate the amount you have to pay to borrow the money from them. The credit card interest rate is usually expressed in yearly rate. This is called annual percentage rate.
The monthly interest rate = \((1+\frac{23}{100})^{(1/12)}\)
= \((1+0.23)^{(1/12)}\)
= 1.0174
Then,
= (1.0174-1) × 100%
Subtract the terms first
= 0.0174 × 100%
Multiply it
= 1.74%
Hence, if a credit card advertises an annual interest rate of 23%, then the equivalent monthly interest rate is 1.74%
Learn more about annual percentage rate here
brainly.com/question/10795072
#SPJ1
m=-3; (1,4) write in slope intercept form
Answer:
y = -3x + 7
Step-by-step explanation:
can someone help plz
Answer:
the Horizontal Aysmptotes is: 4
Range is: (♾,4)
X intercept: None
Y intercept: (0,5)
Decreasing.
Step-by-step explanation:
Hope that will help you
what is the slope of (-3,3) and (-3,13)
Answer:
10
Step-by-step explanation:
y2-y1/x2-x1. subtract the two and reduce the fraction
there are 60 inches in 5 feet.how many inches are in 8 ft?
Answer:
63in
Step-by-step explanation:
In constructing a 95 percent confidence interval, if you increase n to 4n, the width of your confidence interval will (assuming other things remain the same) be:
In constructing a 95 percent confidence interval, if you increase n to 4n, the width of your confidence interval will be reduced by a factor of approximately 1/2, assuming other things remain the same.What is a confidence interval?A confidence interval is a range of values that is likely to contain an unknown population parameter with a certain degree of confidence.
It is frequently used to indicate the accuracy of the estimate. A 95% confidence interval for a population parameter, for example, is a range of values within which we can be 95% confident that the true population parameter falls. Therefore, when you double the sample size, the t value decreases, resulting in a narrower interval. Increasing the sample size four times reduces the width of the confidence interval by a factor of approximately 1/2.
The length of the confidence interval is proportional to the standard error, which is inversely proportional to the square root of the sample size. As a result, increasing the sample size lowers the standard error, which in turn results in a narrower confidence interval.
To know more about confidence visit:-
https://brainly.com/question/29048041
#SPJ11
if each of seven persons in a group shakes hands with each of the other six persons, then a total of forty-two handshakes occurs.
A. True
B. False
Answer:
B. False
Step-by-step explanation:
You want to know if it is true that when each of seven persons in a group shakes hands with each of the other six persons, a total of forty-two handshakes occurs.
HandshakesThe count of 42 handshakes includes every one of 7 shaking hands with each of the remaining 6, without regard to whether they met before.
The count will include A shaking hands with B, and B shaking hands with A. That handshake will be counted twice, so the number 42 overstates the actual number of handshakes by a factor of 2.
It is false that there will be 42 handshakes. (There will be only 21.)
Help would be EXTREMELY appreciated I really need this tomorrow
The value of csc x=-13/5.
Given that the sin x=-5/13 and x∈[3π/2 , 2π).
A trigonometric function is a real function that relates the angle of a right triangle to the ratio of the lengths of the two sides. The basic functions are sin, cos, tan, cot, sec, csc.
The given function is sin x=-5/13.
As we all know that sin x=1/csc x or csc x=1/sin x.
As it is given that x∈[3π/2 , 2π) that means x lies in fourth quadrant and in fourth quadrant sin and csc is negative in it.
So, we will apply this identity csc x=1/sin x, we get
csc x=1/(-5/13)
csc x=-13/5
And it also lies in x∈[3π/2 , 2π).
Hence, the value of csc x when sin x=-5/13 and x∈[3π/2 , 2π) is csc x=-13/5.
Learn more about the trigonometry function from here brainly.com/question/26221138
#SPJ9
Samra's guardians invested money for her into a 529 College Savings Plan, which compounds annually. The growth of the savings plan per year, x, can be represented by the exponential function f(x) = 600(1.04)x. What is the meaning of the y-intercept in the context of the problem?
The principal amount put into the savings plan is $1.04.
The average rate of change that is occurring is 1.04.
The percent rate of change is 600%.
The initial value of the investment is $600.
The meaning of the y-intercept in the context of the problem is D. The initial value of the investment is $600.
How to explain the information?From the information, the growth of the savings plan per year, x, can be represented by the exponential function f(x) = 600(1.04)x.
Here, the meaning of the y-intercept in the context of the problem is the initial value of the investment is $600.
In conclusion, the correct option is D.
Learn more about y intercept on:
brainly.com/question/25722412
#SPJ1
Hello, I am very new to python and I am having trouble with this
problem
The German mathematician Gottfried Leibniz developed the
following method to approximate the value of π:
π = 4(1 - 1/3 + 1/5
To approximate the value of π using the Leibniz method, you can write a Python program that calculates the sum of the series up to a certain number of terms. The more terms you include in the series, the closer the approximation will be to the actual value of π.
The Leibniz method, also known as the Leibniz formula for π, is an infinite series that converges to π/4. The formula is given by:
π = 4(1 - 1/3 + 1/5 - 1/7 + 1/9 - 1/11 + ...)
To approximate π, you can calculate the sum of the series up to a certain number of terms. The more terms you include, the more accurate the approximation will be.
In Python, you can write a program that iterates through the terms of the series and accumulates the sum. Here's an example of how you can implement it:
def approximate_pi(num_terms):
pi = 0
sign = 1
for i in range(1, num_terms*2, 2):
term = sign * (1/i)
pi += term
sign *= -1
return pi * 4
num_terms = 100000 # Choose the number of terms for the approximation
approximation = approximate_pi(num_terms)
In this example, we define the approximate_pi function that takes the number of terms as an argument. The function iterates from 1 to num_terms*2 with a step size of 2, representing the denominators of the series. The sign alternates between positive and negative to include the alternating addition and subtraction. Finally, we return the calculated sum multiplied by 4 to obtain the approximation of π.
By increasing the value of num_terms, you can achieve a more accurate approximation of π. However, keep in mind that the Leibniz method converges slowly, so a large number of terms may be needed for a precise approximation.
To learn more about python
brainly.com/question/30391554
#SPJ11
is a subjective question, hence you have to write your answer in the Text-Field given below. A7008 As a Quality Analyst you are seeing the defects trend by type of defects and are plotting a histogram to do a Pareto analysis. Invent your own data to come up with a Pareto diagram, clearly identifying the top 20% category of defects and once done, deep-dive into the top category to do an Root Cause Analysis and come up with corrective action and preventive action plan. Please state your assumptions clearly at the beginning of your answer. a. Plot a neat histogram on plain paper, and identify the top 20% of the category of defects which contribute to 80% of the total volume of defects. [2 marks] b. Once you identify these top 20% defects, perform a Root Cause Analysis for the Top Contributing Factor using either 5-Why or the Fish-bone diagram method. [4 marks] c. Then, come up with a suitable corrective action plan and a preventive action plan to address the root cause, which should include who will do what
Assumptions: For the purpose of this exercise, let's assume that we are analyzing defects in a manufacturing process. We will invent data for five different categories of defects and their corresponding frequencies. the cumulative percentage for each category, we find that the top 20% category of defects is Category A.
a. Based on the invented data, the histogram analysis reveals the following distribution of defects and their frequencies:
Category A: 50 defects
Category B: 30 defects
Category C: 20 defects
Category D: 15 defects
Category E: 10 defects
To identify the top 20% of the category of defects contributing to 80% of the total volume, we calculate the cumulative frequency. Starting with the category with the highest frequency, we add up the frequencies until we reach 80% of the total. In this case, Category A contributes the highest frequency, and its cumulative frequency is 50. The total number of defects is 125 (50 + 30 + 20 + 15 + 10). By calculating the cumulative percentage for each category, we find that the top 20% category of defects is Category A.
b. Performing a Root Cause Analysis for the Top Contributing Factor (Category A) using the 5-Why method or Fishbone diagram helps determine the underlying causes. We identify potential factors such as equipment malfunction, operator error, insufficient training, or process variability. By asking "why" repeatedly, we dig deeper into each cause to uncover the root cause.
c. Based on the analysis, we develop a corrective action plan and preventive action plan. For example:
Corrective Action Plan: Assign qualified technicians to regularly inspect and maintain the equipment, conduct additional training for operators to enhance their skills, and implement process control measures to reduce variability.
Preventive Action Plan: Establish a preventive maintenance schedule for equipment, implement a comprehensive training program for all operators, and conduct regular process audits to identify and address potential issues proactively.
The corrective and preventive action plans should clearly define the tasks, responsibilities, and timelines. The maintenance department may be responsible for equipment maintenance, the training department for operator training, and the quality department for process audits. Regular monitoring and evaluation of the action plans should be conducted to ensure effectiveness and make any necessary adjustments.
know more about cumulative frequency click here:
https://brainly.com/question/28491523
#SPJ11
I need help!!!!!!!!!!!!!!!!!!!!!!
Answer:
A square
Step-by-step explanation:
L 4.6.3 Test (CST): Linear Equations
me.
OA. y+4= -3(x-3)
OB. y-4=-3(x+3)
OC. y-4=3(x+3)
OD. y+4=3(x-3)
(3,-4)
The correct option is OA. y+4= -3(x-3). L 4.6.3 Test (CST): Linear Equations Solution: We are given that a line passes through (3,-4) and has a slope of -3.
We will use point slope form of line to obtain the equation of liney - y1 = m(x - x1).
Plugging in the values, we get,y - (-4) = -3(x - 3).
Simplifying the above expression, we get y + 4 = -3x + 9y = -3x + 9 - 4y = -3x + 5y = -3x + 5.
This equation is in slope intercept form of line where slope is -3 and y-intercept is 5.The above equation is not matching with any of the options given.
Let's try to put the equation in standard form of line,ax + by = c=> 3x + y = 5
Multiplying all the terms by -1,-3x - y = -5
We observe that option (A) satisfies the above equation of line, therefore correct option is OA. y+4= -3(x-3).
Thus, the correct option is OA. y+4= -3(x-3).
For more question on equation
https://brainly.com/question/17145398
#SPJ8
Hi! I have a question, but when doing algebraic multiplication/division what does it mean when they add a + or a - sign?
example: r - 92 = 215
Answer:
Here is your your answer
Step-by-step explanation:
when ever we solve algebraic multiplication/ division so we have to apply - sign
Hope it is useful
Can someone give me the answers?
Answer:
Step-by-step explanation:
3 1
9 3
15 5
21 7
PLEASE HELP! 43 POINTS
Brenda invited 32 people to her graduation party. If each person will drink about 3 cups of punch, how many gallons of punch does Brenda’s mom need to buy?
How many quarts is this equal to?
Answer:
1 gallon = 16 cups
3 cups per person
32 people
32x3=96/16=6
she needs 6 gallons
HELP ASAP
The table shows input and output values of a function given a rule. Which function
represents the rule?
Input: x, a number; Output: y, 3 less than 2 times x
Input, x: -2 -1 0 1 2
Output, y: -7 -5 -3 -1 1
y=2x+3
y=2x-3
y=-3x+2
y=2x
Answer:I think its A.
Step-by-step explanation:
I'm too lazy too explain you got your answer please be happy
Operations with Scientific Notations!! I need help! Best answer gets the brainliest, please help!
#1: 9 x 10-5 / 9.24 x 10-6
#2: (5 x 10⁵) - (8 x 10⁴)
#3: (5 x 10⁻⁷) - (3 x 10⁻⁹)
#4: (7 x 10⁸) (9 x 10⁶)
#5: (4 x 10⁶) - (2 x 10⁵)
#6: 17 x 108 / 4 x 105
#7: (9 x 10⁴) + (7 x 10)
#8: (6.24 x 10⁻²) + (7.1 x 10⁻³)
#9: (11 x 10⁴) (7 x 10²)
#10: 27 x 10⁴ / 10 x 10⁸
#11: (7 x 10⁷) + (4 x 10⁶)
The problems on scientific notation are solved as below.
What is Scientific Notation?Scientific notation is a way of expressing numbers which are way too large or way too small in a simpler form using exponents or decimals or both.
When adding or subtracting, convert the exponent part same for both numbers.
When multiplying, add the exponents and when dividing, subtract the exponents.
#1: 9 x 10-5 / 9.24 x 10-6 = (9 / 9.24) × 10⁽⁻⁵⁻⁻⁶⁾ = 0.974 × 10¹ = 9.74
#2: (5 x 10⁵) - (8 x 10⁴) = (50 × 10⁴) - (8 × 10⁴) = (50 - 8) × 10⁴ = 42 × 10⁴
#3: (5 x 10⁻⁷) - (3 x 10⁻⁹) = (500 × 10⁻⁹) - (3 × 10⁻⁹) = (500 - 3) × 10⁻⁹ = 497 × 10⁻⁹
#4: (7 x 10⁸) (9 x 10⁶) = (7 × 9) × 10⁸⁺⁶ = 63 × 10¹⁴
#5: (4 x 10⁶) - (2 x 10⁵) = (40 × 10⁵) - (2 × 10⁵) = (40 - 2) × 10⁵ = 38 × 10⁵ = 3800000
#6: 17 x 10⁸ / 4 x 10⁵ = (17 / 4) × 10⁸⁻⁵ = 4.25 × 10³ = 4250
#7: (9 x 10⁴) + (7 x 10) = (9000 × 10) + (7 × 10) = (9000 + 7) × 10 = 90070
#8: (6.24 x 10⁻²) + (7.1 x 10⁻³) = (62.4 × 10⁻³) + (7.1 × 10⁻³) = (62.4 + 7.1) × 10⁻³ = 69.5 × 10⁻³ = 0.0695
#9: (11 x 10⁴) (7 x 10²) = (11 × 7) × 10⁴⁺² = 77 × 10⁶
#10: 27 x 10⁴ / 10 x 10⁸ = (27 / 10) × 10⁴⁻⁸ = 2.7 × 10⁻⁴
#11: (7 x 10⁷) + (4 x 10⁶) = (70 × 10⁶) + (4 × 10⁶) = (70 + 4) × 10⁶ = 74 × 10⁶
Hence the problems related to scientific notation are solved.
Solve more problem with Scientific Notation here :
https://brainly.com/question/18073768
#SPJ1
if a group of students having an average age of 16 years joined a class, the average age of all the students in the class reduces from 18 years to 17 years. what is the ratio of the number of students who joined the class to the number of students who were initially in the class?
The ratio of the number of students who joined the class to the number of students who were initially in the class is 2:1.
What is age ratio?Age ratio refers to the comparison of the number of individuals in different age groups in a population. It is often expressed as a ratio or percentage of the number of people in a particular age group to the total population or the number of people in another age group.
Given by the question.
Now, when the group of students with an average age of 16 years joins the class, the average age of all the students becomes 17 years. This means that the total age of all the students in the class would now be 17 times the total number of students.
Let "y" be the number of students who joined the class. Then, the total number of students in the class after they joined would be x + y.
So, we have two equations:
18x = 17(x+y) (because the total age of all the students remains the same before and after the group joins)
16y = (x+y)*1 (because the average age of the new group is 16)
Simplifying the above equations, we get:
x = 17y
x = 34y
Equating both the equations, we get:
17y = 34y
y = 2
To learn more about age ratio:
https://brainly.com/question/29963980
#SPJ1
Check if (1,6) is the solution to the system shown below.
Answer:
NO it is not
Step-by-step explanation:
To check that, we will need to substitute the value of each of the coordinates in each of the equations
10x-y = -6
substitute 1 for x and 6 for y
we have this as;
10(1)-6 = 4
Outrightly, we can see that the given point is not a solution to the given system of equations
however, let us confirm for the second
We have this as;
-10(1) + 5(6)
= -10 + 30 = 20
so we can confirm that it does not work for any of the two and our answer is NO
a study was performed to test the effectiveness of a new treatment for migraines. there were 125 patients in the study. of these, 60 were randomly assigned to receive the standard treatment and 65 to receive the new treatment. all were instructed to take the medication at the onset of their next migraine and to notice if the pain of their migraine felt reduced after half an hour. fifty patients receiving the new treatment reported reduced pain, compared with only 40 of the patients receiving the standard treatment. the results seem to favor the new treatment, but could the difference just be due to chance?
Answer:
The difference in the effectiveness of the new treatment versus the standard treatment may be due to chance, and further studies may be needed to determine the effectiveness of the new treatment.
Step-by-step explanation:
To determine if the difference in the number of patients reporting reduced pain is statistically significant, we can use a hypothesis test.
The null hypothesis is that there is no difference between the two treatments in terms of the proportion of patients who report reduced pain after half an hour. The alternative hypothesis is that there is a difference between the two treatments.
We can use a two-sample proportion test to compare the proportions of patients reporting reduced pain in the two groups.
The test statistic is calculated as:
z = (p1 - p2) / sqrt(\(\sqrt{(p_hat * (1 - p_hat) / n1) + (p_hat * (1 - p_hat) / n2)}\))
where p1 and p2 are the proportions of patients in the two groups reporting reduced pain, p_ hat is the pooled proportion (calculated as the total number of patients reporting reduced pain divided by the total number of patients), and n1 and n2 are the sample sizes.
In this case, we have:
p1 = 50/65 = 0.769
p2 = 40/60 = 0.667
p_ hat = (50 + 40) / (65 + 60) = 0.672
n1 = 65
n2 = 60
Substituting these values into the formula, we get:
z = (0.769 - 0.667) / sqrt((0.672 * (1 - 0.672) / 65) + (0.672 * (1 - 0.672) / 60))
= 1.412
Using a standard normal distribution table or calculator, we can find the probability of obtaining a z-value of 1.412 or greater, assuming the null hypothesis is true. This is the p-value of the test.
The p-value turns out to be approximately 0.078, which is greater than the commonly used significance level of 0.05. This means that there is not enough evidence to reject the null hypothesis and we cannot conclude that the difference in the number of patients reporting reduced pain is statistically significant.
Therefore, the difference in the effectiveness of the new treatment versus the standard treatment may be due to chance, and further studies may be needed to determine the effectiveness of the new treatment.
To know more about effectiveness refer here
https://brainly.com/question/30664353#
#SPJ11
Can someone help me quick please ?
Let's look at the answer choices to see what the questions wants us to do with the data
⇒ essentially they compare Carol's mean or median to herself or to
Anna's mean or media
⇒ therefore we should calculate Carol's mean, median and Anna's
mean, median
Before we start calculating, let's define what is the mean and median
Mean: it is basically the average which is the sum of all numbers in the set divided by the number of numbers in that setMedian: it is the middle number of the set when the set is ordered from lowest to highestLet's calculate:
Carol's mean,median⇒ Carol's data ordered: 20, 30, 45, 60, 70
\(Mean = \frac{20+30+45+60+70}{5} = \frac{225}{5}=45\\ \\Median: 45\)
Anna's mean, median⇒ Anna's data ordered: 30, 35, 45, 60, 65
\(Mean = \frac{30+ 35+ 45+ 60+ 65}{5} =\frac{235}{5} =47\\\\Median: 45\\\)
Now using the data, let's compare:
first choice: Carol's mean is equal to her median--> correct
second choice: Carol's mean is not greater than Anna's mean--> incorrect
third choice: Anna's median isnot greater than her mean --> incorrect
fourth choice: Anna's mean is not equal to Carol's mean --> incorrect
Hope that helps!
George and his friend Jane buy copies of the same book on the internet. George pays $16.95 and jane pays £11.99 on a day when the exchange rate is $1 = £ 0.626.
calculate, in dollars, how much more Jane pays.
Answer:
$19.15
Step-by-step explanation:
$1=0.626
$1/0.626=£1
$11.99=$11.99/0.626
=$19.15
Answer:
Step-by-step explanation:
£ 0.626 =$ 1
£1 = $ \(\frac{1}{0.625}\)
£ 11.99 = \(11.99*\frac{1}{0.625}= \frac{11.99*1000}{0.625*1000}\\\\\)
\(= \frac{11990}{626}\\\\\)
= $ 19.15
19.15 - 16.95 = $2.2
Jane pays $2.20 more dollars that George
Golden Fleece Management stock is expected to pay a dividend of $3.11 in 1 year. The stock is currently priced at $65.62, is expected to be priced at $72.00 in 1 year, and is expected to be priced at $75.99 in 2 years. What is the dividend in 2 years expected to be for Golden Fleece Management stock? The stock's dividend is paid annually and the next dividend is expected in 1 year. a. An amount less than $3.18 or an amount greater than $7.87 b. An amount equal to or greater than $5.01 but less than $6.90 c. An amount equal to or greater than $6.90 but less than $7.87 d. An amount equal to or greater than $3.18 but less than $3.44 e. An amount equal to or greater than $3.44 but less than $5.01
The answer to the question can be obtained as follows:Given, Dividend after 1 year, D1 = $3.11Price of stock today, P0 = $65.62Expected price after 1 year, P1 = $72.00Expected price after 2 years, P2 = $75.99Dividend after 2 years, D2 = ?The dividend discount model can be used to solve the problem since the expected dividend is paid annually.
D1 = D0(1 + g) where g is the growth rate of the dividend.Using the formula for the present value of a stock, we have
P0 = D0 / (1 + r)^1 + D1 / (1 + r)^2 + P1 / (1 + r)^2
where r is the expected return on the stock.Substituting the values,P0 = 3.11 / (1 + r)^1 + D2 / (1 + r)^2 + 72.00 / (1 + r)^2....(1)Solving for the expected dividend in 2 years,D2 = P0(1 + r)^2 - 3.11(1 + r) - 72(1 + r)^2 / (1 + r)^2....(2)We can now substitute the values given to obtainD2 = 75.99(1 + r)^2 - 3.11(1 + r) - 72(1 + r)^2 / (1 + r)^2From this, we get a quadratic equation to solve for r.
75.99(1 + r)^2 - 3.11(1 + r) - 72 = 0
Solving the equation we get two roots: -1.13 and 0.72 since the expected rate of return cannot be negative, we use the positive value i.e. 0.72.Substituting the value of r into (2), we obtain:D2 = $6.16Therefore, the expected dividend after 2 years is $6.16. Option C is, therefore, correct.
To know more about quadratic equation visit :
https://brainly.com/question/30098550
#SPJ11
The quantities X and Y are proportional
The quantities X and Y are proportional as y = rx then the constant of proportionality r is 1/9.
What is constant of proportionality?Firstly, there is proportional relationship.
Two values x and y are said to be in a proportional relationship if x=ky, where x and y are variables and k is a constant.
The constant k is called constant of proportionality.
WE have been given quantities X and Y are proportional
y = rx
Where r is constant of proportionality.
Then x = 45 and y = 5
Thus,
y = rx
5 = 45r
Now divide by 45 on both side;
r = 5/45
r = 1/9
Therefore, the constant of proportionality r is 1/9.
Learn more about directly inversely proportional relationship variable here:
https://brainly.com/question/13082482
#SPJ1
a cyclist travels 25 kilometers at 20 km/h and then a further 80 kilometers at 25 km/h what is the total time taken?
Answer: 4 9/20
Step-by-step explanation: Since it is 20km/h, and it is 25km, you can find 1/4th of 20, which is 5, and add it because 25 is 1 1/4 of 20, he traveled 1 1/4 hours on the first session, and then he goes and does 80km. same thing, exept it is a lot bigger than 25km/h, and the fact that the cyclist is faster by 5km/h. so then, you can multiply to get the closest number which is not greater than 80, which is 75. so we know it is around 3 hours, so since it is 5 km, and 25 km is faster than 20km, it is going to be 1/5th of an hour extra. so 3 1/5 hours + 1 1/4 hours is 4 9/20. I cannot simplify because it is already a mixed number, and there is no LCM between 9 and 20. so that's it.
Sorry if I extremely overexplained.
Which of the following is a key property of the quadratic parent function?
OA. It is in quadrants III and IV.
OB. Its vertex is at the origin.
C. It is not a parabola.
OD. It is not a function.
The correct property of the quadratic parent function is given by:
B. Its vertex is at the origin.
What is the quadratic parent function?The quadratic parent function is modeled by:
y = x².
It has the vertex at the origin and increases to the left and to the right, into quadrants I and II, forming a function that is graphed by a parabola.
Hence the correct option regarding a property of the quadratic parent function is given by:
B. Its vertex is at the origin.
More can be learned about the quadratic parent function at https://brainly.com/question/12980287
#SPJ1
i need help pleaseee its due in 10 min
what is 244 + 6
pls give me relevent answer
Answer: 1
4
244+6=300 [its a simple addition right?] 3
brainliest? J
E
N
<3 :)
Pls help! giving thanks. What is the slope of the line (picture above)
Answer:
\(slope = 1 \)
Step-by-step explanation:
We can see that the graph cuts the x axis at , (-1,0) and y axis at (0,1) . So we can find the slope as ,
\(\implies slope = \dfrac{y_2-y_1}{x_2-x_1}\\\\\implies slope = \dfrac{ 1-0}{0+1} \\\\\implies\boxed{\boxed{ slope = 1 }}\)